A Very Illegal Position












4














It's been a while since I've done a chess puzzle, so here's one!



This position is illegal in quite a lot of ways:



enter image description here



It's illegal in at least 10 ways. The first one to find all of the ways this position is illegal (it's 10 or more, I know exactly how many) gets the check and +1 from me.



Good luck and happy puzzling!










share|improve this question
























  • i assume you mean for us to find the illegal ways and not the legal ways
    – AHKieran
    2 hours ago






  • 1




    @AHKieran fixed that little typo :P
    – Excited Raichu
    2 hours ago
















4














It's been a while since I've done a chess puzzle, so here's one!



This position is illegal in quite a lot of ways:



enter image description here



It's illegal in at least 10 ways. The first one to find all of the ways this position is illegal (it's 10 or more, I know exactly how many) gets the check and +1 from me.



Good luck and happy puzzling!










share|improve this question
























  • i assume you mean for us to find the illegal ways and not the legal ways
    – AHKieran
    2 hours ago






  • 1




    @AHKieran fixed that little typo :P
    – Excited Raichu
    2 hours ago














4












4








4


1





It's been a while since I've done a chess puzzle, so here's one!



This position is illegal in quite a lot of ways:



enter image description here



It's illegal in at least 10 ways. The first one to find all of the ways this position is illegal (it's 10 or more, I know exactly how many) gets the check and +1 from me.



Good luck and happy puzzling!










share|improve this question















It's been a while since I've done a chess puzzle, so here's one!



This position is illegal in quite a lot of ways:



enter image description here



It's illegal in at least 10 ways. The first one to find all of the ways this position is illegal (it's 10 or more, I know exactly how many) gets the check and +1 from me.



Good luck and happy puzzling!







chess






share|improve this question















share|improve this question













share|improve this question




share|improve this question








edited 2 hours ago

























asked 2 hours ago









Excited Raichu

5,8802863




5,8802863












  • i assume you mean for us to find the illegal ways and not the legal ways
    – AHKieran
    2 hours ago






  • 1




    @AHKieran fixed that little typo :P
    – Excited Raichu
    2 hours ago


















  • i assume you mean for us to find the illegal ways and not the legal ways
    – AHKieran
    2 hours ago






  • 1




    @AHKieran fixed that little typo :P
    – Excited Raichu
    2 hours ago
















i assume you mean for us to find the illegal ways and not the legal ways
– AHKieran
2 hours ago




i assume you mean for us to find the illegal ways and not the legal ways
– AHKieran
2 hours ago




1




1




@AHKieran fixed that little typo :P
– Excited Raichu
2 hours ago




@AHKieran fixed that little typo :P
– Excited Raichu
2 hours ago










2 Answers
2






active

oldest

votes


















3














1.




Both kings are in check.




2.




The black king is in check by both the Rc3 and Bg4.




3.




The white bishop on b1 can only arrive there via a2 and c2, and there are white pawns on a2 and c2 which can't have moved.




4.




Black has nine pawns.




5.




White has two light-squared bishops; one must have been promoted but White has still eight pawns.




6.




White's pawns seem to have captured three times (b-pawn to d5, d-pawn to e3), but Black's only missing piece is a single knight. Also, the f7 pawn can only get there via captures (unless the black f6 pawn somehow dropped out of thin air, see 4.)




7.




There's no way for the black rook which started on h8 to escape (to either b6 or a1).






I'm not sure I can find more; for example,




it will be hard for the Ra1 to reach that square, but once you assume the Bb1 materialized out of thin air (see 3.), it's not a problem anymore.




Also,




the black pawns are 7 columns 'away' from their home squares, but if you assume the g3 has been dropped there, only 4 captures are required to account for the a- and b-pawns, and there are four white pieces missing (two knights, a rook and the queen).




A pitfall:




the white king can be put in check this way; Black's last move could have been Nh4+ (discovered check)




Note that




some of these irregularities could happen during a game of bughouse.







share|improve this answer























  • rook h8 could have been captured by a white knight, though
    – Bass
    2 hours ago










  • True, but that would require another promoted black pawn.
    – Glorfindel
    2 hours ago



















1















1. bishop in g4 in check

2. rook in c3 in check

3. bishop in f3 in check

4. rook in a1 cant get there

5. 9 black pawns

6. pawn in a3 cant get there

7. pawn in g3 cant get there

8. pawn in f7 cant get there

9. 2 white bishop on white squares

10. pawn in e3 cant get there

11. bishop in b1 cant get there







share|improve this answer























  • Your #4 and #12 are the same. The A3 pawn could have come from the C column by capturing twice, same goes for G3 from the D column, although since black is only missing 2 pieces, one of these must be illegal.
    – Nuclear Wang
    1 hour ago











Your Answer





StackExchange.ifUsing("editor", function () {
return StackExchange.using("mathjaxEditing", function () {
StackExchange.MarkdownEditor.creationCallbacks.add(function (editor, postfix) {
StackExchange.mathjaxEditing.prepareWmdForMathJax(editor, postfix, [["$", "$"], ["\\(","\\)"]]);
});
});
}, "mathjax-editing");

StackExchange.ready(function() {
var channelOptions = {
tags: "".split(" "),
id: "559"
};
initTagRenderer("".split(" "), "".split(" "), channelOptions);

StackExchange.using("externalEditor", function() {
// Have to fire editor after snippets, if snippets enabled
if (StackExchange.settings.snippets.snippetsEnabled) {
StackExchange.using("snippets", function() {
createEditor();
});
}
else {
createEditor();
}
});

function createEditor() {
StackExchange.prepareEditor({
heartbeatType: 'answer',
autoActivateHeartbeat: false,
convertImagesToLinks: false,
noModals: true,
showLowRepImageUploadWarning: true,
reputationToPostImages: null,
bindNavPrevention: true,
postfix: "",
imageUploader: {
brandingHtml: "Powered by u003ca class="icon-imgur-white" href="https://imgur.com/"u003eu003c/au003e",
contentPolicyHtml: "User contributions licensed under u003ca href="https://creativecommons.org/licenses/by-sa/3.0/"u003ecc by-sa 3.0 with attribution requiredu003c/au003e u003ca href="https://stackoverflow.com/legal/content-policy"u003e(content policy)u003c/au003e",
allowUrls: true
},
noCode: true, onDemand: true,
discardSelector: ".discard-answer"
,immediatelyShowMarkdownHelp:true
});


}
});














draft saved

draft discarded


















StackExchange.ready(
function () {
StackExchange.openid.initPostLogin('.new-post-login', 'https%3a%2f%2fpuzzling.stackexchange.com%2fquestions%2f78081%2fa-very-illegal-position%23new-answer', 'question_page');
}
);

Post as a guest















Required, but never shown

























2 Answers
2






active

oldest

votes








2 Answers
2






active

oldest

votes









active

oldest

votes






active

oldest

votes









3














1.




Both kings are in check.




2.




The black king is in check by both the Rc3 and Bg4.




3.




The white bishop on b1 can only arrive there via a2 and c2, and there are white pawns on a2 and c2 which can't have moved.




4.




Black has nine pawns.




5.




White has two light-squared bishops; one must have been promoted but White has still eight pawns.




6.




White's pawns seem to have captured three times (b-pawn to d5, d-pawn to e3), but Black's only missing piece is a single knight. Also, the f7 pawn can only get there via captures (unless the black f6 pawn somehow dropped out of thin air, see 4.)




7.




There's no way for the black rook which started on h8 to escape (to either b6 or a1).






I'm not sure I can find more; for example,




it will be hard for the Ra1 to reach that square, but once you assume the Bb1 materialized out of thin air (see 3.), it's not a problem anymore.




Also,




the black pawns are 7 columns 'away' from their home squares, but if you assume the g3 has been dropped there, only 4 captures are required to account for the a- and b-pawns, and there are four white pieces missing (two knights, a rook and the queen).




A pitfall:




the white king can be put in check this way; Black's last move could have been Nh4+ (discovered check)




Note that




some of these irregularities could happen during a game of bughouse.







share|improve this answer























  • rook h8 could have been captured by a white knight, though
    – Bass
    2 hours ago










  • True, but that would require another promoted black pawn.
    – Glorfindel
    2 hours ago
















3














1.




Both kings are in check.




2.




The black king is in check by both the Rc3 and Bg4.




3.




The white bishop on b1 can only arrive there via a2 and c2, and there are white pawns on a2 and c2 which can't have moved.




4.




Black has nine pawns.




5.




White has two light-squared bishops; one must have been promoted but White has still eight pawns.




6.




White's pawns seem to have captured three times (b-pawn to d5, d-pawn to e3), but Black's only missing piece is a single knight. Also, the f7 pawn can only get there via captures (unless the black f6 pawn somehow dropped out of thin air, see 4.)




7.




There's no way for the black rook which started on h8 to escape (to either b6 or a1).






I'm not sure I can find more; for example,




it will be hard for the Ra1 to reach that square, but once you assume the Bb1 materialized out of thin air (see 3.), it's not a problem anymore.




Also,




the black pawns are 7 columns 'away' from their home squares, but if you assume the g3 has been dropped there, only 4 captures are required to account for the a- and b-pawns, and there are four white pieces missing (two knights, a rook and the queen).




A pitfall:




the white king can be put in check this way; Black's last move could have been Nh4+ (discovered check)




Note that




some of these irregularities could happen during a game of bughouse.







share|improve this answer























  • rook h8 could have been captured by a white knight, though
    – Bass
    2 hours ago










  • True, but that would require another promoted black pawn.
    – Glorfindel
    2 hours ago














3












3








3






1.




Both kings are in check.




2.




The black king is in check by both the Rc3 and Bg4.




3.




The white bishop on b1 can only arrive there via a2 and c2, and there are white pawns on a2 and c2 which can't have moved.




4.




Black has nine pawns.




5.




White has two light-squared bishops; one must have been promoted but White has still eight pawns.




6.




White's pawns seem to have captured three times (b-pawn to d5, d-pawn to e3), but Black's only missing piece is a single knight. Also, the f7 pawn can only get there via captures (unless the black f6 pawn somehow dropped out of thin air, see 4.)




7.




There's no way for the black rook which started on h8 to escape (to either b6 or a1).






I'm not sure I can find more; for example,




it will be hard for the Ra1 to reach that square, but once you assume the Bb1 materialized out of thin air (see 3.), it's not a problem anymore.




Also,




the black pawns are 7 columns 'away' from their home squares, but if you assume the g3 has been dropped there, only 4 captures are required to account for the a- and b-pawns, and there are four white pieces missing (two knights, a rook and the queen).




A pitfall:




the white king can be put in check this way; Black's last move could have been Nh4+ (discovered check)




Note that




some of these irregularities could happen during a game of bughouse.







share|improve this answer














1.




Both kings are in check.




2.




The black king is in check by both the Rc3 and Bg4.




3.




The white bishop on b1 can only arrive there via a2 and c2, and there are white pawns on a2 and c2 which can't have moved.




4.




Black has nine pawns.




5.




White has two light-squared bishops; one must have been promoted but White has still eight pawns.




6.




White's pawns seem to have captured three times (b-pawn to d5, d-pawn to e3), but Black's only missing piece is a single knight. Also, the f7 pawn can only get there via captures (unless the black f6 pawn somehow dropped out of thin air, see 4.)




7.




There's no way for the black rook which started on h8 to escape (to either b6 or a1).






I'm not sure I can find more; for example,




it will be hard for the Ra1 to reach that square, but once you assume the Bb1 materialized out of thin air (see 3.), it's not a problem anymore.




Also,




the black pawns are 7 columns 'away' from their home squares, but if you assume the g3 has been dropped there, only 4 captures are required to account for the a- and b-pawns, and there are four white pieces missing (two knights, a rook and the queen).




A pitfall:




the white king can be put in check this way; Black's last move could have been Nh4+ (discovered check)




Note that




some of these irregularities could happen during a game of bughouse.








share|improve this answer














share|improve this answer



share|improve this answer








edited 2 hours ago

























answered 2 hours ago









Glorfindel

13.3k34982




13.3k34982












  • rook h8 could have been captured by a white knight, though
    – Bass
    2 hours ago










  • True, but that would require another promoted black pawn.
    – Glorfindel
    2 hours ago


















  • rook h8 could have been captured by a white knight, though
    – Bass
    2 hours ago










  • True, but that would require another promoted black pawn.
    – Glorfindel
    2 hours ago
















rook h8 could have been captured by a white knight, though
– Bass
2 hours ago




rook h8 could have been captured by a white knight, though
– Bass
2 hours ago












True, but that would require another promoted black pawn.
– Glorfindel
2 hours ago




True, but that would require another promoted black pawn.
– Glorfindel
2 hours ago











1















1. bishop in g4 in check

2. rook in c3 in check

3. bishop in f3 in check

4. rook in a1 cant get there

5. 9 black pawns

6. pawn in a3 cant get there

7. pawn in g3 cant get there

8. pawn in f7 cant get there

9. 2 white bishop on white squares

10. pawn in e3 cant get there

11. bishop in b1 cant get there







share|improve this answer























  • Your #4 and #12 are the same. The A3 pawn could have come from the C column by capturing twice, same goes for G3 from the D column, although since black is only missing 2 pieces, one of these must be illegal.
    – Nuclear Wang
    1 hour ago
















1















1. bishop in g4 in check

2. rook in c3 in check

3. bishop in f3 in check

4. rook in a1 cant get there

5. 9 black pawns

6. pawn in a3 cant get there

7. pawn in g3 cant get there

8. pawn in f7 cant get there

9. 2 white bishop on white squares

10. pawn in e3 cant get there

11. bishop in b1 cant get there







share|improve this answer























  • Your #4 and #12 are the same. The A3 pawn could have come from the C column by capturing twice, same goes for G3 from the D column, although since black is only missing 2 pieces, one of these must be illegal.
    – Nuclear Wang
    1 hour ago














1












1








1







1. bishop in g4 in check

2. rook in c3 in check

3. bishop in f3 in check

4. rook in a1 cant get there

5. 9 black pawns

6. pawn in a3 cant get there

7. pawn in g3 cant get there

8. pawn in f7 cant get there

9. 2 white bishop on white squares

10. pawn in e3 cant get there

11. bishop in b1 cant get there







share|improve this answer















1. bishop in g4 in check

2. rook in c3 in check

3. bishop in f3 in check

4. rook in a1 cant get there

5. 9 black pawns

6. pawn in a3 cant get there

7. pawn in g3 cant get there

8. pawn in f7 cant get there

9. 2 white bishop on white squares

10. pawn in e3 cant get there

11. bishop in b1 cant get there








share|improve this answer














share|improve this answer



share|improve this answer








edited 1 hour ago

























answered 2 hours ago









pirate

491115




491115












  • Your #4 and #12 are the same. The A3 pawn could have come from the C column by capturing twice, same goes for G3 from the D column, although since black is only missing 2 pieces, one of these must be illegal.
    – Nuclear Wang
    1 hour ago


















  • Your #4 and #12 are the same. The A3 pawn could have come from the C column by capturing twice, same goes for G3 from the D column, although since black is only missing 2 pieces, one of these must be illegal.
    – Nuclear Wang
    1 hour ago
















Your #4 and #12 are the same. The A3 pawn could have come from the C column by capturing twice, same goes for G3 from the D column, although since black is only missing 2 pieces, one of these must be illegal.
– Nuclear Wang
1 hour ago




Your #4 and #12 are the same. The A3 pawn could have come from the C column by capturing twice, same goes for G3 from the D column, although since black is only missing 2 pieces, one of these must be illegal.
– Nuclear Wang
1 hour ago


















draft saved

draft discarded




















































Thanks for contributing an answer to Puzzling Stack Exchange!


  • Please be sure to answer the question. Provide details and share your research!

But avoid



  • Asking for help, clarification, or responding to other answers.

  • Making statements based on opinion; back them up with references or personal experience.


Use MathJax to format equations. MathJax reference.


To learn more, see our tips on writing great answers.





Some of your past answers have not been well-received, and you're in danger of being blocked from answering.


Please pay close attention to the following guidance:


  • Please be sure to answer the question. Provide details and share your research!

But avoid



  • Asking for help, clarification, or responding to other answers.

  • Making statements based on opinion; back them up with references or personal experience.


To learn more, see our tips on writing great answers.




draft saved


draft discarded














StackExchange.ready(
function () {
StackExchange.openid.initPostLogin('.new-post-login', 'https%3a%2f%2fpuzzling.stackexchange.com%2fquestions%2f78081%2fa-very-illegal-position%23new-answer', 'question_page');
}
);

Post as a guest















Required, but never shown





















































Required, but never shown














Required, but never shown












Required, but never shown







Required, but never shown

































Required, but never shown














Required, but never shown












Required, but never shown







Required, but never shown







Popular posts from this blog

Understanding the information contained in the Deep Space Network XML data?

Ross-on-Wye

Eastern Orthodox Church